Difference between revisions of "2014 AMC 12B Problems/Problem 24"

(Solution)
m (Solution 4)
 
(29 intermediate revisions by 8 users not shown)
Line 8: Line 8:
 
\textbf{(E) }421\qquad</math>
 
\textbf{(E) }421\qquad</math>
  
==Solution==
+
==Video Solution by Punxsutawney Phil==
 +
https://www.youtube.com/watch?v=1-2vT_GIceA
  
 +
== Solution 1 ==
 +
Let <math>BE=a</math>, <math>AD=b</math>, and <math>AC=CE=BD=c</math>. Let <math>F</math> be on <math>AE</math> such that <math>CF \perp AE</math>.
 +
<asy>
 +
size(200);
 +
defaultpen(linewidth(0.4)+fontsize(10));
 +
pen s = linewidth(0.8)+fontsize(8);
 +
 +
pair O,A,B,C,D,E0,F;
 +
O=origin;
 +
A= dir(198);
 +
path c = CR(O,1);
 +
real r = 0.13535;
 +
B = IP(c, CR(A,3*r));
 +
C = IP(c, CR(B,10*r));
 +
D = IP(c, CR(C,3*r));
 +
E0 = OP(c, CR(D,10*r));
 +
F = foot(C,A,E0);
 +
 +
dot("$A$", A, A-O);
 +
dot("$B$", B, B-O);
 +
dot("$C$", C, C-O);
 +
dot("$D$", D, D-O);
 +
dot("$E$", E0, E0-O);
 +
dot("$F$", F, F-C);
 +
label("$c$",A--C,S);
 +
label("$c$",E0--C,W);
 +
label("$7$",F--E0,S);
 +
label("$7$",F--A,S);
 +
label("$3$",A--B,2*W);
 +
label("$10$",B--C,2*N);
 +
label("$3$",C--D,2*NE);
 +
label("$10$",D--E0,E);
 +
draw(A--B--C--D--E0--A, black+0.8);
 +
 +
draw(CR(O,1), s);
 +
draw(A--C--E0, royalblue);
 +
draw(C--F, royalblue+dashed);
 +
draw(rightanglemark(E0,F,C,2));
 +
MA("\theta",A,B,C,0.075);
 +
MA("\pi-\theta",C,E0,A,0.1);
 +
</asy>
 +
In <math>\triangle CFE</math> we have <math>\cos\theta = -\cos(\pi-\theta)=-7/c</math>. We use the [[Law of Cosines]] on <math>\triangle ABC</math> to get <math>60\cos\theta = 109-c^2</math>.  Eliminating <math>\cos\theta</math> we get <math>c^3-109c-420=0</math> which factorizes as
 +
<cmath>(c+7)(c+5)(c-12)=0.</cmath>Discarding the negative roots we have <math>c=12</math>. Thus <math>BD=AC=CE=12</math>. For <math>BE=a</math>, we use Ptolemy's theorem on cyclic quadrilateral <math>ABCE</math> to get <math>a=44/3</math>. For <math>AD=b</math>, we use [[Ptolemy's theorem]] on cyclic quadrilateral <math>ACDE</math> to get <math>b=27/2</math>.
 +
 +
The sum of the lengths of the diagonals is <math>12+12+12+\tfrac{44}{3}+\tfrac{27}{2} = \tfrac{385}{6}</math> so the answer is <math>385 + 6 = \fbox{\textbf{(D) }391}</math>
 +
 +
== Solution 2 ==
 
Let <math>a</math> denote the length of a diagonal opposite adjacent sides of length <math>14</math> and <math>3</math>, <math>b</math> for sides <math>14</math> and <math>10</math>, and <math>c</math> for sides <math>3</math> and <math>10</math>. Using Ptolemy's Theorem on the five possible quadrilaterals in the configuration, we obtain:
 
Let <math>a</math> denote the length of a diagonal opposite adjacent sides of length <math>14</math> and <math>3</math>, <math>b</math> for sides <math>14</math> and <math>10</math>, and <math>c</math> for sides <math>3</math> and <math>10</math>. Using Ptolemy's Theorem on the five possible quadrilaterals in the configuration, we obtain:
  
Line 16: Line 64:
 
c^2 &= 3a+100 \\
 
c^2 &= 3a+100 \\
 
c^2 &= 10b+9 \\
 
c^2 &= 10b+9 \\
14c &= 30+ab \\
+
ab &= 30+14c \\
 
ac &= 3c+140\\
 
ac &= 3c+140\\
 
bc &= 10c+42
 
bc &= 10c+42
Line 46: Line 94:
 
</cmath>
 
</cmath>
  
<math>c</math>, being a length, must be positive, implying that <math>c=12</math>. Plugging this back into equations <math>(1)</math> and <math>(2)</math> we find that <math>a = \frac{44}{3}</math> and <math>b= \frac{135}{10}=\frac{27}{2}</math>.
+
Or similarly into equation <math>(5)</math> to check:
 +
 
 +
<cmath>
 +
\begin{align*}
 +
\frac{c^2-9}{10}c &= 10c+42\\
 +
\frac{c^3-9c}{10} &= 10c + 42\\
 +
c^3-9c &= 100c + 420\\
 +
c^3-109c-420 &=0\\
 +
(c-12)(c+7)(c+5)&=0
 +
\end{align*}
 +
</cmath>
 +
 
 +
<math>c</math>, being a length, must be positive, implying that <math>c=12</math>. In fact, this is reasonable, since <math>10+3\approx 12</math> in the pentagon with apparently obtuse angles. Plugging this back into equations <math>(1)</math> and <math>(2)</math> we find that <math>a = \frac{44}{3}</math> and <math>b= \frac{135}{10}=\frac{27}{2}</math>.
 +
 
 +
We desire <math>3c+a+b = 3\cdot 12 + \frac{44}{3} + \frac{27}{2} = \frac{216+88+81}{6}=\frac{385}{6}</math>, so it follows that the answer is <math>385 + 6 = \fbox{\textbf{(D) }391}</math>
 +
 
 +
==Solution 3 (Ptolemy's but Quicker)==
 +
 
 +
Let us set <math>x</math> to be <math>AC=BD=CE</math> and <math>y</math> to be <math>BE</math> and <math>z</math> to be <math>AD</math>. It follow from applying [[Ptolemy's Theorem]] on <math>ABCD</math> to get <math>x^2=9+10z</math>. Applying Ptolemy's on <math>ACDE</math> gives <math>xz=42+10x</math>; and applying Ptolemy's on <math>BCDE</math> gives <math>x^2=100+3y</math>. So, we have the have the following system of equations:
 +
 
 +
<cmath>
 +
\begin{align}
 +
x^2 &= 9+10z \\
 +
x^2 &= 100+3y \\
 +
xz &= 42+10x
 +
\end{align}
 +
</cmath>
 +
 
 +
From <math>(3)</math>, we have <math>42=(z-10)x</math>. Isolating the x gives <math>x=\dfrac{42}{z-10}</math>. By setting <math>(1)</math> and <math>(2)</math> equal, we have <math>x^2=9+10z=100+3y</math>. Manipulating it gives <math>3y=10z-91</math>. Finally, plugging back into <math>(2)</math> gives <math>x^2=100+10z-91=10z+9</math>. Plugging in the <math>x=\dfrac{42}{z-10}</math> as well gives
 +
 
 +
<cmath>
 +
\begin{align*}
 +
\left(\frac{42}{z-10}\right)^2 &= 10z+9\\
 +
10z^3 - 191z^2 + 820z + 900 &= 1764\\
 +
10z^3 - 191z^2 + 820z - 864 &= 0\\
 +
(5z-8)(2z-27)(z-4) &=0
 +
\end{align*}
 +
</cmath>
 +
 
 +
It is impossible for <math>z<10</math> for <math>x<0</math>; that means <math>z=\frac{27}{2}</math>. That means <math>x = 12</math> and <math>y = \frac{44}{3}</math>.
  
We desire <math>3c+a+b = 3\cdot 12 + \frac{44}{3} + \frac{27}{2} = \frac{385}{6}</math>, so it follows that the answer is <math>385 + 6 = 391</math>.
+
Thus, the sum of all diagonals is <math>3x+y+z = 3\cdot 12 + \frac{44}{3} + \frac{27}{2} = 385/6</math>, which implies our answer is <math>m+n = 385+6 = \fbox{391 \textbf{(D)}}</math>.
 +
 
 +
~sml1809
 +
 
 +
==Solution 4==
 +
[[File:2014AMC12BProblem24Solution4.png|center|400px]]
 +
Let <math>BE = a</math>, <math>AC = CE = BD = b</math>
 +
 
 +
By [[Ptolemy's theorem]] for quadrilateral <math>ABCE</math>, <math>AB \cdot CE + BC \cdot AE = BE \cdot AC</math>, <math>3b + 140 = ab</math>, <math>a = 3 + \frac{140}{b}</math>
 +
 
 +
By [[Ptolemy's theorem]] for quadrilateral <math>BCDE</math>, <math>CD \cdot BE + BC \cdot DE = BD \cdot CE</math>, <math>3a + 100 = b^2</math>
 +
 
 +
<math>3(3 + \frac{140}{b}) + 100 = b^2</math>, <math>b^3 - 109 b -420 = 0</math>, <math>(b-12)(b+7)(b+5) = 0</math>, <math>b = 12</math>
 +
 
 +
<math>a = 3 + \frac{140}{12} = \frac{44}{3}</math>
 +
 
 +
By [[Ptolemy's theorem]] for quadrilateral <math>ABDE</math>, <math>AE \cdot BD + AB \cdot DE = AD \cdot BE</math>, <math>AD \cdot a = 14b + 30</math>, <math>AD = \frac{27}{2}</math>
 +
 
 +
<math>\frac{m}{n} = 12 + 12 + 12 + \frac{44}{3} + \frac{27}{2} = \frac{385}{6}</math>, <math>385 + 6 = \boxed{\textbf{(D) }391}</math>
 +
 
 +
~[https://artofproblemsolving.com/wiki/index.php/User:Isabelchen isabelchen]
  
 
== See also ==
 
== See also ==
 
{{AMC12 box|year=2014|ab=B|num-b=23|num-a=25}}
 
{{AMC12 box|year=2014|ab=B|num-b=23|num-a=25}}
 +
 +
[[Category:Intermediate Geometry Problems]]
 
{{MAA Notice}}
 
{{MAA Notice}}

Latest revision as of 08:09, 1 January 2023

Problem

Let $ABCDE$ be a pentagon inscribed in a circle such that $AB = CD = 3$, $BC = DE = 10$, and $AE= 14$. The sum of the lengths of all diagonals of $ABCDE$ is equal to $\frac{m}{n}$, where $m$ and $n$ are relatively prime positive integers. What is $m+n$ ?

$\textbf{(A) }129\qquad \textbf{(B) }247\qquad \textbf{(C) }353\qquad \textbf{(D) }391\qquad \textbf{(E) }421\qquad$

Video Solution by Punxsutawney Phil

https://www.youtube.com/watch?v=1-2vT_GIceA

Solution 1

Let $BE=a$, $AD=b$, and $AC=CE=BD=c$. Let $F$ be on $AE$ such that $CF \perp AE$. [asy] size(200); defaultpen(linewidth(0.4)+fontsize(10)); pen s = linewidth(0.8)+fontsize(8);  pair O,A,B,C,D,E0,F; O=origin; A= dir(198); path c = CR(O,1); real r = 0.13535; B = IP(c, CR(A,3*r)); C = IP(c, CR(B,10*r)); D = IP(c, CR(C,3*r)); E0 = OP(c, CR(D,10*r)); F = foot(C,A,E0);  dot("$A$", A, A-O); dot("$B$", B, B-O); dot("$C$", C, C-O); dot("$D$", D, D-O); dot("$E$", E0, E0-O); dot("$F$", F, F-C); label("$c$",A--C,S); label("$c$",E0--C,W); label("$7$",F--E0,S); label("$7$",F--A,S); label("$3$",A--B,2*W); label("$10$",B--C,2*N); label("$3$",C--D,2*NE); label("$10$",D--E0,E); draw(A--B--C--D--E0--A, black+0.8);  draw(CR(O,1), s); draw(A--C--E0, royalblue); draw(C--F, royalblue+dashed); draw(rightanglemark(E0,F,C,2)); MA("\theta",A,B,C,0.075); MA("\pi-\theta",C,E0,A,0.1); [/asy] In $\triangle CFE$ we have $\cos\theta = -\cos(\pi-\theta)=-7/c$. We use the Law of Cosines on $\triangle ABC$ to get $60\cos\theta = 109-c^2$. Eliminating $\cos\theta$ we get $c^3-109c-420=0$ which factorizes as \[(c+7)(c+5)(c-12)=0.\]Discarding the negative roots we have $c=12$. Thus $BD=AC=CE=12$. For $BE=a$, we use Ptolemy's theorem on cyclic quadrilateral $ABCE$ to get $a=44/3$. For $AD=b$, we use Ptolemy's theorem on cyclic quadrilateral $ACDE$ to get $b=27/2$.

The sum of the lengths of the diagonals is $12+12+12+\tfrac{44}{3}+\tfrac{27}{2} = \tfrac{385}{6}$ so the answer is $385 + 6 = \fbox{\textbf{(D) }391}$

Solution 2

Let $a$ denote the length of a diagonal opposite adjacent sides of length $14$ and $3$, $b$ for sides $14$ and $10$, and $c$ for sides $3$ and $10$. Using Ptolemy's Theorem on the five possible quadrilaterals in the configuration, we obtain:

\begin{align} c^2 &= 3a+100 \\ c^2 &= 10b+9 \\ ab &= 30+14c \\ ac &= 3c+140\\ bc &= 10c+42 \end{align}

Using equations $(1)$ and $(2)$, we obtain:

\[a = \frac{c^2-100}{3}\]

and

\[b = \frac{c^2-9}{10}\]

Plugging into equation $(4)$, we find that:

\begin{align*} \frac{c^2-100}{3}c &= 3c + 140\\ \frac{c^3-100c}{3} &= 3c + 140\\ c^3-100c &= 9c + 420\\ c^3-109c-420 &=0\\ (c-12)(c+7)(c+5)&=0 \end{align*}

Or similarly into equation $(5)$ to check:

\begin{align*} \frac{c^2-9}{10}c &= 10c+42\\ \frac{c^3-9c}{10} &= 10c + 42\\ c^3-9c &= 100c + 420\\ c^3-109c-420 &=0\\ (c-12)(c+7)(c+5)&=0 \end{align*}

$c$, being a length, must be positive, implying that $c=12$. In fact, this is reasonable, since $10+3\approx 12$ in the pentagon with apparently obtuse angles. Plugging this back into equations $(1)$ and $(2)$ we find that $a = \frac{44}{3}$ and $b= \frac{135}{10}=\frac{27}{2}$.

We desire $3c+a+b = 3\cdot 12 + \frac{44}{3} + \frac{27}{2} = \frac{216+88+81}{6}=\frac{385}{6}$, so it follows that the answer is $385 + 6 = \fbox{\textbf{(D) }391}$

Solution 3 (Ptolemy's but Quicker)

Let us set $x$ to be $AC=BD=CE$ and $y$ to be $BE$ and $z$ to be $AD$. It follow from applying Ptolemy's Theorem on $ABCD$ to get $x^2=9+10z$. Applying Ptolemy's on $ACDE$ gives $xz=42+10x$; and applying Ptolemy's on $BCDE$ gives $x^2=100+3y$. So, we have the have the following system of equations:

\begin{align} x^2 &= 9+10z \\ x^2 &= 100+3y \\ xz &= 42+10x \end{align}

From $(3)$, we have $42=(z-10)x$. Isolating the x gives $x=\dfrac{42}{z-10}$. By setting $(1)$ and $(2)$ equal, we have $x^2=9+10z=100+3y$. Manipulating it gives $3y=10z-91$. Finally, plugging back into $(2)$ gives $x^2=100+10z-91=10z+9$. Plugging in the $x=\dfrac{42}{z-10}$ as well gives

\begin{align*} \left(\frac{42}{z-10}\right)^2 &= 10z+9\\ 10z^3 - 191z^2 + 820z + 900 &= 1764\\ 10z^3 - 191z^2 + 820z - 864 &= 0\\ (5z-8)(2z-27)(z-4) &=0 \end{align*}

It is impossible for $z<10$ for $x<0$; that means $z=\frac{27}{2}$. That means $x = 12$ and $y = \frac{44}{3}$.

Thus, the sum of all diagonals is $3x+y+z = 3\cdot 12 + \frac{44}{3} + \frac{27}{2} = 385/6$, which implies our answer is $m+n = 385+6 = \fbox{391 \textbf{(D)}}$.

~sml1809

Solution 4

2014AMC12BProblem24Solution4.png

Let $BE = a$, $AC = CE = BD = b$

By Ptolemy's theorem for quadrilateral $ABCE$, $AB \cdot CE + BC \cdot AE = BE \cdot AC$, $3b + 140 = ab$, $a = 3 + \frac{140}{b}$

By Ptolemy's theorem for quadrilateral $BCDE$, $CD \cdot BE + BC \cdot DE = BD \cdot CE$, $3a + 100 = b^2$

$3(3 + \frac{140}{b}) + 100 = b^2$, $b^3 - 109 b -420 = 0$, $(b-12)(b+7)(b+5) = 0$, $b = 12$

$a = 3 + \frac{140}{12} = \frac{44}{3}$

By Ptolemy's theorem for quadrilateral $ABDE$, $AE \cdot BD + AB \cdot DE = AD \cdot BE$, $AD \cdot a = 14b + 30$, $AD = \frac{27}{2}$

$\frac{m}{n} = 12 + 12 + 12 + \frac{44}{3} + \frac{27}{2} = \frac{385}{6}$, $385 + 6 = \boxed{\textbf{(D) }391}$

~isabelchen

See also

2014 AMC 12B (ProblemsAnswer KeyResources)
Preceded by
Problem 23
Followed by
Problem 25
1 2 3 4 5 6 7 8 9 10 11 12 13 14 15 16 17 18 19 20 21 22 23 24 25
All AMC 12 Problems and Solutions

The problems on this page are copyrighted by the Mathematical Association of America's American Mathematics Competitions. AMC logo.png